Đến nội dung

Hình ảnh

Topic: [LTDH] Mỗi ngày hai bất đẳng thức.


  • Chủ đề bị khóa Chủ đề bị khóa
Chủ đề này có 215 trả lời

#61
tritanngo99

tritanngo99

    Đại úy

  • Điều hành viên THPT
  • 1644 Bài viết

Tiếp theo: 

Bài 29: Cho $x,y,z$ là các số dương thỏa mãn: $(\frac{x}{y})^4+(\frac{y}{z})^4+\frac{z}{x}=\frac{x}{z}+2$. Tìm GTLN của:

$P=\frac{2y^2}{x^2+y^2}+\frac{2z^2}{y^2+z^2}-\frac{3z}{2x+z}$

Bài 30: Cho $x,y,z\in (0;1]$ thỏa mãn: $x+y-z\ge 1$. Tìm GTNN của:

$B=\frac{x}{y+z}+\frac{y}{z+x}+\frac{z}{xy+z^2}$.

Bài 30*: Giả sử $x,y,z\ge 0$ thỏa mãn: $x+y+z=3$. Hãy tìm GTNN của biểu thức: $P=x^4+8y^4+64z^4$

Ps: Do quochungtran là thành viên mới, chưa đọc rõ nội quy nên xem đây là trường hợp đặc biệt của Topic, mình sẽ lấy bài của bạn quochungtran xem như bài 30*. Và hi vọng lần sau bạn chú ý hơn:))

(Ngoài ra mình xin sửa đề một chút cho dễ tính toán)


Bài viết đã được chỉnh sửa nội dung bởi tritanngo99: 21-08-2016 - 09:11


#62
NTA1907

NTA1907

    Thượng úy

  • Thành viên
  • 1014 Bài viết

Tiếp theo: 

Bài 30*: Giả sử $x,y,z\ge 0$ thỏa mãn: $x+y+z=3$. Hãy tìm GTNN của biểu thức: $P=x^4+8y^4+64z^4$

Giả định $x=a, y=b, z=c$

Áp dụng AM-GM ta có:

$x^{4}+a^{4}+a^{4}+a^{4}\geq 4xa^{3}$

$8(y^{4}+b^{4}+b^{4}+b^{4})\geq 32yb^{3}$

$64(z^{4}+c^{4}+c^{4}+c^{4})\geq 256zc^{3}$

Cộng các bất đẳng thức trên vế theo vế ta được:

$P\geq 4xa^{3}+32yb^{3}+256zc^{3}-3a^{4}-24b^{4}-192c^{4}$

Ta tìm $a,b,c$ thoả mãn hệ sau:

$\left\{\begin{matrix} &a+b+c=3 \\ &4a^{3}=32b^{3}=256z^{3} \end{matrix}\right.$

$\Leftrightarrow \left\{\begin{matrix} &a=\frac{12}{7} & \\ &b=\frac{6}{7} & \\ &c=\frac{3}{7} & \end{matrix}\right.$

$\Rightarrow P\geq \frac{5184}{343}$

Dấu = xảy ra$\Leftrightarrow x=\frac{12}{7}, y=\frac{6}{7}, z=\frac{3}{7}$


Vũ trụ không có biên trong không gian, không có bắt đầu và kết thúc trong thời gian và chẳng có việc gì cho đấng sáng thế phải làm ở đây cả.

 


#63
quochungtran

quochungtran

    Lính mới

  • Thành viên mới
  • 5 Bài viết

Đây là lời giải bài 30*)

Chọn các số a,b,c dương và a+b+c = 3, theo bất đẳng thức Holder

(x+ 2y+ 3z4)(a4 + 2b4 + 3c4)3 $\geq$ (a3x + 2b3y + 3c3z)4.

Chọn a,b,c sao cho a3 = 2b3 = 3c = k3. khi đó :

x+ 2y+ 3z$\geq$ $\frac{k^{12}(x+y+z)^{4}}{a^{4}+2b^{4}+3c^{4}} $ = $\frac{(3k^{3})^{4}}{(a^{4}+2b^{4}+3c^{4})^{3}}$

xét điều kiện đẳng thức thì

                                        $\frac{x}{a}$ =  $\frac{y}{b}$ =  $\frac{z}{c}$ = $\frac{x+y+z}{a+b+c}$ = 1

 

              Vậy ta có a+b+c = 3 , a3 = 2b3 = 3c = k3  => x=a=k, y=b=$\sqrt[3]{2}$ k , z=c= $\sqrt[3]{3}$ k , vs k= $\frac{3}{1+\sqrt[3]{2}+\sqrt[3]{3}}$

 

Ps: mình thấy lời giải NTA 1907 chưa chặc chẽ lắm


Bài viết đã được chỉnh sửa nội dung bởi quochungtran: 21-08-2016 - 16:06

~O)  ~O)  ~O)  :excl:  :excl:  :excl:  ~O)  ~O)  ~O)  :ukliam2:


#64
tritanngo99

tritanngo99

    Đại úy

  • Điều hành viên THPT
  • 1644 Bài viết

Dưới đây là lời giải bài 29,bài 30 và bài 30*:

Lời giải bài 29: 

Đặt $t=\frac{z}{x}$. Theo $AM-GM$ ta có:

$2+\frac{x}{z}=\frac{z}{x}+(\frac{x}{y})^4+(\frac{y}{z})^4\ge \frac{z}{x}+\frac{2z^2}{x^2}\iff t+\frac{2}{t^2}\le 2+\frac{1}{t}\iff t^3-2t^2-t+2\le 0\iff t\in [1;2]$.

Viết lại $P$ ta có: $P=4-(\frac{2x^2}{x^2+y^2}+\frac{2y^2}{y^2+z^2}+\frac{3z}{2x+z})=4-Q$.

Ta đi tìm GTNN của Q.

Ta có: $\frac{x^2}{x^2+y^2}+\frac{y^2}{y^2+z^2}=\frac{1}{1+(\frac{y}{x})^2}+\frac{1}{1+(\frac{z}{y})^2}\ge \frac{2}{1+\frac{z}{x}}\text{(do} \frac{z}{x}\ge 1)$.

(Áp dụng BDT quen thuộc: $\frac{1}{1+a^2}+\frac{1}{1+b^2}\ge \frac{2}{1+ab}\forall ab\ge 1$).

Suy ra: $Q\ge \frac{4}{1+\frac{z}{x}}+\frac{3*\frac{z}{x}}{2+\frac{z}{x}}=\frac{4}{1+t}+\frac{3t}{t+2}$.

Xét hàm số: $f(t)=\frac{4}{t+1}+\frac{3t}{t+2};t\in [1;2]$.

Ta tìm được $GTNN$ của $Q$ bằng $\frac{17}{6}$.Đạt được khi $t=2$.

Vậy $GTLN$ của $P$ là $\frac{7}{6}$.Đẳng thức xảy ra khi:$\left\{\begin{matrix} x=a\\y=\sqrt{2}a\\z=2a  \end{matrix}\right.$

Lời giải bài 30:

Theo giả thiết, ta có: $x+y-z\ge 1\implies x+y\ge 1+z$.

Ta có: $B=\frac{x}{y+z}+\frac{y}{z+x}+\frac{z}{xy+z^2}=\frac{x^2}{x(y+z)}+\frac{y^2}{y(z+x)}+\frac{z}{xy+z^2}$.

Áp dụng BDT BCS ta có: $B\ge \frac{(x+y)^2}{2xy+z(x+y)}+\frac{z}{xy+z^2}(1)$.

Đặt $t=x+y,$ theo giả thiết ta có: $1+z\le t\le 2;xy\le \frac{t^2}{4}(2)$.

Thoe $(1),(2)$, ta suy ra được: $B\ge \frac{2t^2}{t^2+2zt}+\frac{4z}{t^2+z^2}=f(t)$.

Xét hàm số:$f(t)$ trên $[1+z;2]$ ta có:

$f'(t)=4zt[\frac{t}{(t^2+2zt)^2}-\frac{2}{(t^2+4z^2)^2}]$.

Mặt khác do $t\ge z+1;z\le 1$ nên $2zt\ge 4z^2$

$\implies \frac{t}{(t^2+2zt)^2}\le \frac{2}{(t^2+4z^2)^2}$

$\implies f(t)$ nghịch biến với mọi $t\in [z+1;2]\implies f(t)\ge f(2)=\frac{2}{1+z}+\frac{z}{z^2+1}=g(z)$.

Ta sẽ khảo sát hàm số: $g(z)$ trên $(0;1]$. Ta có: $g'(z)=-\frac{2}{(1+z)^2}+\frac{1-z^2}{(z^2+1)^2}\le 0\forall z\in (0;1]$

$\implies g(z)\ge g(1)=\frac{3}{2}$.

Vậy $Min_B=\frac{3}{2}$. Dấu $=$ xảy ra khi $x=y=z=1$.

Lời giải bài 30*:

Ta có: $2P=2x^4+16y^4+128z^4$.

Ta sẽ đi chứng minh:

$2x^4\ge \frac{13824x}{343}-\frac{124416}{2401}(1)$

$16y^4\ge \frac{13824y}{343}-\frac{62208}{2401}(2)$.

$128z^4\ge \frac{13824z}{343}-\frac{31104}{2401}(3)$

Thật vậy:

$(1)\iff (7x-12)^2(98x^2+336x+864)\ge 0\forall x$

$(2)\iff (7x-6)^2(784y^2+1344y+1728)\ge 0\forall y$

$(3)\iff (7z-3)^2(6272z^2+5376z+3456)\ge 0\forall z$.

Cộng $(1),(2),(3)$ vế theo vế ta được:

$2P\ge \frac{13824}{343}(x+y+z)-\frac{31104}{343}=\frac{10368}{343}$

$\iff P\ge \frac{5184}{343}$.

Vậy $P_{min}=\frac{5184}{343}$.Dấu $=$ xảy ra tại $x=\frac{12}{7},y=\frac{6}{7},z=\frac{3}{7}$.


Bài viết đã được chỉnh sửa nội dung bởi tritanngo99: 22-08-2016 - 08:04


#65
tritanngo99

tritanngo99

    Đại úy

  • Điều hành viên THPT
  • 1644 Bài viết

Tiếp theo:

Bài 31: Cho $a,b,c>0$ thỏa mãn: $\frac{4a}{b}(1+\frac{2c}{b})+\frac{b}{a}(1+\frac{c}{a})=6$. Tìm GTNN của biểu thức:

$B=\frac{bc}{a(b+2c)}+2[\frac{ac}{b(a+c)}+\frac{ab}{c(2a+b)}]$.

Bài 32: Cho $3$ số thực dương $x,y,z$ thỏa mãn: $3xy+yz+2zx=6$. Tìm GTLN của biểu thức:

$P=\frac{1}{x^2+1}+\frac{4}{y^2+4}+\frac{3z}{9+z^2}$



#66
phamngochung9a

phamngochung9a

    Sĩ quan

  • Điều hành viên THPT
  • 480 Bài viết

Tiếp theo:

Bài 31: Cho $a,b,c>0$ thỏa mãn: $\frac{4a}{b}(1+\frac{2c}{b})+\frac{b}{a}(1+\frac{c}{a})=6$. Tìm GTNN của biểu thức:

$B=\frac{bc}{a(b+2c)}+2[\frac{ac}{b(a+c)}+\frac{ab}{c(2a+b)}]$.

Bài 32: Cho $3$ số thực dương $x,y,z$ thỏa mãn: $3xy+yz+2zx=6$. Tìm GTLN của biểu thức:

$P=\frac{1}{x^2+1}+\frac{4}{y^2+4}+\frac{3z}{9+z^2}$

Bài 31:

Đặt $\left\{\begin{matrix} 2a=x & & \\ b=y & & \\ 2c=z & & \end{matrix}\right.$, giả thiết trở thành: 

 

$\frac{2x}{y}\left (1+\frac{z}{y} \right )+\frac{2y}{x}\left ( 1+\frac{z}{x} \right )=6\\\Rightarrow 3x^{2}y^{2}= x^{3}y+xy^{3}+z\left ( x^{3}+y^{3} \right )\geq 2x^{2}y^{2}+xy\left ( x+y \right )z\\\Rightarrow xy\geq z\left ( x+y \right )$

 

Khi đó:

 

$B=\frac{xz}{y\left ( x+z \right )}+\frac{yz}{x\left ( y+z \right )}+\frac{2xy}{z\left ( x+y \right )}\\\geq z.\frac{\left ( x+y \right )^{2}}{xy\left ( x+y+2z \right )}+\frac{2}{3}.\frac{xy}{z\left ( x+y \right )}+\frac{4}{3}.\frac{xy}{z\left ( x+y \right )}\\\geq 2\sqrt{\frac{2}{3}.\frac{x+y}{x+y+2z}}+\frac{4}{3}.\frac{z\left ( x+y \right )}{z\left ( x+y \right )}\\=2\sqrt{\frac{2}{3}.\frac{1}{1+2.\frac{z}{x+y}}}+\frac{4}{3}$

 

Vì $xy\geq z\left ( x+y \right )\Rightarrow z\left ( x+y \right )\leq \frac{\left ( x+y \right )^{2}}{4}\Rightarrow \frac{x+y}{z}\geq 4$

 

Vậy:

 

$B\geq 2\sqrt{\frac{2}{3}.\frac{1}{1+2.\frac{z}{x+y}}}+\frac{4}{3}\geq \frac{8}{3}$

 

Bài 32:

 

Từ giả thiết, ta có: $x.\frac{y}{2}+\frac{y}{2}.\frac{z}{3}+\frac{z}{3}.x=1$

 

Đặt $\left\{\begin{matrix} x=\tan\frac{A}{2} & & \\ \frac{y}{2}=\tan\frac{B}{2} & & \\ \frac{z}{3}=\tan\frac{C}{2} & & \end{matrix}\right.$ với $A,B,C$ là ba góc của một tam giác. Ta có:

 

$P=\frac{1}{1+\tan^{2}\frac{A}{2}}+\frac{4}{4+4\tan^{2}\frac{B}{2}}+\frac{9\tan^{2}\frac{C}{2}}{9+9\tan^{2}\frac{C}{2}}\\=\cos^{2}\frac{A}{2}+\cos^{2}\frac{B}{2}+\sin^{2}\frac{C}{2}\\=\frac{3}{2}+\frac{1}{2}\left ( \cos A+\cos B-\cos C \right )\\=\frac{3}{2}+\frac{1}{2}\left [ 2\cos\left ( \frac{A+B}{2} \right ).\cos \left ( \frac{A-B}{2} \right )-\cos C \right ]\\\leq \frac{3}{2}+\frac{1}{2}\left ( 2\cos \frac{A+B}{2}-\cos C \right )\\=\frac{3}{2}+\frac{1}{2}\left ( 2\cos \frac{C}{2}-2\cos^{2}\frac{C}{2}+1 \right )\leq \frac{9}{4}$

 

Vậy $\max P=\frac{9}{4}$


Bài viết đã được chỉnh sửa nội dung bởi phamngochung9a: 22-08-2016 - 14:31


#67
tritanngo99

tritanngo99

    Đại úy

  • Điều hành viên THPT
  • 1644 Bài viết

Bạn phamngochung9a giải hai bài 31 và 32 quá chuẩn rồi nên mình xin đề xuất hai bài tiếp theo:

Bài 33: Cho $a,b,c$ là các số thực dương thỏa mãn: $abc=1$. Chứng minh rằng:

$\frac{a}{a^2+3}+\frac{b}{b^2+3}+\frac{c}{c^2+3}\le \frac{3}{4}$.

Bài 34: Cho $a,b,c$ là các số thực dương thỏa mãn: $a^2+b^2+c^2+2abc=1$. Chứng minh rằng:

$a^2+b^2+c^2\ge 4(a^2b^2+b^2c^2+c^2a^2)$ 


Bài viết đã được chỉnh sửa nội dung bởi tritanngo99: 24-08-2016 - 09:50


#68
tritanngo99

tritanngo99

    Đại úy

  • Điều hành viên THPT
  • 1644 Bài viết

Dưới đây là lời giải bài 33 và bài 34:

Lời giải bài 33:

Ta có: $abc=1\iff ln(a)+ln(b)+ln(c)=0$

Đặt: $(ln(a);ln(b);ln(c))\rightarrow (x;y;z)\implies x+y+z=0$.

Khi đó: $VT=\frac{e^x}{(e^x)^2+3}+\frac{e^y}{(e^y)^2+3}+\frac{e^z}{(e^z)^2+3}$.

Xét $f(x)=\frac{e^x}{(e^x)^2+3}$.

Sử dụng phương pháp tiếp tuyến: Ta có: $f(x)=\frac{e^x}{(e^x)^2+3}\le \frac{1}{8}x+\frac{1}{4}$.

Làm tương tự rồi cộng lại ta có: $f(x)+f(y)+f(z)\le \frac{1}{8}(x+y+z)+\frac{3}{4}=\frac{3}{4}$.

Lời giải bài 34:

Với giả thiết đã cho luôn tồn tại một tam giác $ABC$ sao cho $a=cosA;b=cosB;c=cosC$. Khi ấy ta có:

$cos^2A+cos^2B+cos^2C+2cosAcosBcosC=1$.

Bất đẳng thức cần được chứng minh viết lại: $cos^2A+cos^2B+cos^2C\ge 4(cos^2Acos^2B+cos^2Bcos^2C+cos^2Ccos^2A)$.

Ta thấy: $cos^2A=cot^2Asin^2A=\frac{cot^2A}{cot^2A+1}$.

Đặt: $(x;y;z)\rightarrow (cotA,cotB,cotC)$

Ta đi chứng minh: $\frac{x^2}{x^+1}+\frac{y^2}{y^2+1}+\frac{z^2}{z^+1}\ge 4(\frac{x^2y^2}{(x^2+1)(y^2+1)}+\frac{y^2z^2}{(y^2+1)(z^2+1)}+\frac{z^2x^2}{(z^2+1)(x^2+1)})(1)$.

Ta có giả thiết mới: $xy+yz+zx=1$ và $x^2+1=(x+y)(x+z)$.

Bất đẳng thức $(1)$ được viết dưới dạng:

$\frac{x^2}{(x+y)(x+z)}+\frac{y^2}{(y+z)(y+z)}+\frac{z^2}{(z+x)(z+y)}\ge 4(\sum \frac{x^2y^2}{(x+y)^2(y+z)(z+x)})$.

$\iff x^2(y+z)+y^2(z+x)+z^2(x+y)\ge 4(\frac{x^2y^2}{x+y}+\frac{y^2z^2}{y+z}+\frac{z^2x^2}{x+z})$

 Bây giờ,áp dụng $BCS$, ta có: $\frac{4x^2y^2}{x+y}\le \frac{x^2y^2}{x}+\frac{x^2y^2}{y}$.

Dấu $=$ xảy ra khi $a=b=c=\frac{1}{2}$.



#69
tritanngo99

tritanngo99

    Đại úy

  • Điều hành viên THPT
  • 1644 Bài viết

Tiếp theo: 

Bài 35: Cho $x,y,z>0$ và thỏa mãn: $2x+4y+7z=xyz$. Tìm GTNN của $P=x+y+z$

Bài 36: Cho $x,y$ là hai số thực dương thỏa mãn: $x^2+y^2=2$. Chứng minh rằng: $\frac{x^3}{y^2}+\frac{9y^2}{x+2y}\ge 4$. 



#70
nguyenduy287

nguyenduy287

    Thượng sĩ

  • Thành viên
  • 256 Bài viết

Tiếp theo: 

Bài 35: Cho $x,y,z>0$ và thỏa mãn: $2x+4y+7z=xyz$. Tìm GTNN của $P=x+y+z$

Bài 36: Cho $x,y$ là hai số thực dương thỏa mãn: $x^2+y^2=2$. Chứng minh rằng: $\frac{x^3}{y^2}+\frac{9y^2}{x+2y}\ge 4$. 

Bài 35 là bài chọn đội tuyển việt nam năm 2001

Áp dụng bđt cauchy mở rộng ta có 

$P=x+y+z=6\frac{x}{6}+5\frac{y}{5}+4\frac{z}{4}\geq 15(\frac{x}{6})^{\frac{6}{15}}(\frac{y}{5})^{\frac{5}{15}}(\frac{z}{4})^{\frac{4}{15}}$

Và $1=\frac{1}{yz}+\frac{2}{xz}+\frac{7}{2xy}=6\frac{1}{yz}+10\frac{1}{5zx}+14\frac{1}{4xy}\geq 30(\frac{1}{6yz})^{\frac{6}{30}}(\frac{1}{5zx})^{\frac{10}{30}}(\frac{1}{4xy})^{\frac{14}{30}}$

Từ đó $P^2.1\geq (\frac{15}{2})^2=>p\geq \frac{15}{2}$

Dấu bằng xảy ra khi và chỉ khi $\left\{\begin{matrix}\frac{x}{6}=\frac{y}{5}=\frac{z}{4} \\ 2x+4y+7z=2xyz \end{matrix}\right.=>\left\{\begin{matrix}x=3 \\ y=\frac{5}{2} \\ z=2 \end{matrix}\right.$


  "DÙ BẠN NGHĨ BẠN CÓ THỂ HAY BẠN KHÔNG THỂ, BẠN ĐỀU ĐÚNG "

                                                                                               -Henry Ford -

  

 

 

 

 


#71
tritanngo99

tritanngo99

    Đại úy

  • Điều hành viên THPT
  • 1644 Bài viết

Dưới đây là lời giải bài 35 và bài 36:

Lời giải bài 35: Bạn nguyenduy287 đã giải chuẩn rồi. Đây là một bài kĩ thuật khá độc đáo và mang tính ý tưởng rất cao.

Lời giải bài 36:

Cách 1: Biến đổi tương đương ta cần chứng minh:

$x^3(x+2y)+9y^4\ge 4y^2(x+2y)$.

Ta đồng bậc hóa đẳng thức: $x^3(x+2y)+9y^4\ge 2y^2(x+2y)\sqrt{2(x^2+y^2)}$.

$\iff [x^3(x+2y)+9y^4]^2\ge [2y^2(x+2y)\sqrt{2(x^2+y^2)}]^2$

Khai triển trực tiếp và rút gọn ta được:

$x^8+49y^8+4x^6y^2+4x^3y^5+10x^4y^4-40x^2y^6-32xy^7\ge 0$.

Đến đây, đặt: $t=\frac{x}{y}>0$, ta được: $t^8+4t^7+4t^6+10t^4+4t^3-40t^2-32t+49\ge 0$.

$\iff (t-1)^2(t^6+6t^5+14t^4+24t^3+43t^2+66t+49)\ge 0(TRUE)$.

Cách 2: Áp dụng BDT $BCS$ ta có: $(\frac{x^3}{y^2}+\frac{9y^2}{x+2y})[x+(x+2y)]\ge (\frac{x^2}{y}+3y)^2$

Lại do: $x^2+y^2=2$ nên $\frac{x^2}{y}+3y=\frac{2-y^2}{y}+3y=2(y+\frac{1}{y})\ge 4$

Thêm vào đó, dễ dàng chứng minh được: $x+x+2y=2(x+y)\le 4$.

Nên suy ra được: $\frac{x^3}{y^2}+\frac{9y^2}{x+2y}\ge 4$.

Đẳng thức xảy ra khi: $x=y=1$.

Cách 3: Ta có: $\frac{x^3}{y^2}+\frac{9y^2}{x+2y}\ge \frac{x^3}{y^2}+\frac{9y^2}{x+y^2+1}$

Vì $x^2+y^2=2$ nên ta có: $\frac{x^3}{y^2}+\frac{9y^2}{x+y^2+1}=\frac{x^3}{1-x^2}+\frac{9(2-x^2)}{x+3-x^2}$

Bây giờ, ta cần chỉ ra rằng;

$\frac{x^3}{2-x^2}+\frac{9(2-x^2)}{x+3-x^2}\ge 4\iff \frac{(x-1)^2(x^3-4x^2-16x-12)}{(2-x^2)(x+3-x^2)}\le 0(TRUE..DO...0<x<\sqrt{2})$

Từ đây ta có DPCM.

Ps: Đây là ba phương pháp thường dùng trong chứng minh bất đẳng thức. Trong đó: Kĩ thuật một tuy đơn giản nhưng biến đổi tốn rất nhiều thời gian.


Bài viết đã được chỉnh sửa nội dung bởi tritanngo99: 25-08-2016 - 10:12


#72
tritanngo99

tritanngo99

    Đại úy

  • Điều hành viên THPT
  • 1644 Bài viết

Tiếp theo: 

Bài 37: Cho $a,b,c>0$ thỏa mãn: $a+b+c=1$. Chứng minh rằng:

$\frac{a}{a+bc}+\frac{b}{b+ac}+\frac{\sqrt{abc}}{c+ab}\le 1+\frac{3\sqrt{3}}{4}$.

Bài 38: Cho $a,b,c\le0$ thỏa mãn: $a+b+c=1$. Chứng minh rằng:

$\frac{ab+bc+ca}{a^2b^2+b^2c^2+c^2a^2}\ge 8(a^2+b^2+c^2)$.


Bài viết đã được chỉnh sửa nội dung bởi tritanngo99: 26-08-2016 - 07:05


#73
royal1534

royal1534

    Trung úy

  • Điều hành viên THCS
  • 773 Bài viết

Bài 38: Cho $a,b,c>0$ thỏa mãn: $a+b+c=1$. Chứng minh rằng:

$\frac{ab+bc+ca}{a^2b^2+b^2c^2+c^2a^2}\ge 8(a^2+b^2+c^2)$.

Hình như đề phải là $a,b,c \geq 0$ anh nhỉ ...

Đổi biến $(a+b+c,abc,ab+bc+ca)=(p,r,q)$

Ta có đánh giá: $q^2 \geq 3pr=3r>2r \rightarrow q^2-2r>0$

Với $p=1$. BĐT cần chứng minh tương đương với:

$\frac{q}{q^2-2r} \geq 8(1-2q)$

$\Leftrightarrow 16r+\frac{q(4q-1)^2}{1-2q} \geq 0$ (Đúng vì $q \leq \frac{p^2}{3}=\frac{1}{3}<\frac{1}{2}$)

Ta có đpcm. Dấu '=' xảy ra khi $a=b=\frac{1}{2},c=0$


Bài viết đã được chỉnh sửa nội dung bởi royal1534: 25-08-2016 - 23:57


#74
tritanngo99

tritanngo99

    Đại úy

  • Điều hành viên THPT
  • 1644 Bài viết

Dưới đây là lời giải bài 37 và bài 38:

Lời giải bài 37: Ta có: $A=\frac{1}{1+\frac{bc}{a}}+\frac{1}{1+\frac{ca}{b}}+\frac{1}{\sqrt{\frac{c}{ab}}+\sqrt{\frac{ab}{c}}}$

Đặt: $(x;y;z)\rightarrow (\sqrt{\frac{bc}{a}};\sqrt{\frac{ca}{b}};\sqrt{\frac{ab}{c}})$ thì ta có: $xy+yz+zx=1$.

$\implies P=\frac{1}{1+x^2}+\frac{1}{1+y^2}+\frac{x}{1+x^2}$.

Đến đây dấu hiệu lượng giác khá rõ ràng:

Tồn tại $\triangle{ABC}$ sao cho: $tan\frac{A}{2}=x,tan\frac{B}{2}=y,tan\frac{C}{2}=z$.

Khi đó: $A=\frac{sinA+cosB+cosC}{2}+1$

Ta có: $cosB+cosC=2cos(\frac{B+C}{2})cos(\frac{B-C}{2})\le 2cos(\frac{B+C}{2})=2sin(\frac{A}{2})$.

Ta sẽ chứng minh:$\frac{sinA+2sin\frac{A}{2}}{2}\le \frac{3\sqrt{3}}{4}$

$\iff sinA+2sin\frac{A}{2}\le \frac{3\sqrt{3}}{2}$

$\iff 2t\sqrt{1-t^2}+2t\le \frac{3\sqrt{3}}{2}\iff (t-\frac{\sqrt{3}}{2})(16t^2+16\sqrt{3}t+36)\ge 0(TRUE)$.

Vậy ta có: $A\le 1+\frac{3\sqrt{3}}{4}$.Dấu $=$ xảy ra khi $A=\frac{2\pi}{3},B=C=\frac{\pi}{6}\iff a=\sqrt{3},b=c=2-\sqrt{3}$

Lời giải bài 38: royal1534 giải theo phương pháp $p,q,r$ là quá chuẩn rồi, ở đây mình trình bày một lời giải khác cũng tương tự ý tưởng trên như sau:

 Đặt $t=ab+bc+ca\implies 0\le t\le \frac{(a+b+c)^2}{3}=\frac{1}{3}$.

Áp dụng điều kiện: $a+b+c=1$, ta có:

$\frac{t}{t^2-2abc(a+b+c)}\ge 8[(a+b+c)^2-2(ab+bc+ca)]$

$\iff \frac{t}{t^2-2abc}\ge 8(1-2t)$

Để ý: $\frac{t}{t^2-2abc}\ge \frac{t}{t^2}=\frac{1}{t}$.

Do đó ta chỉ cần chứng minh: $\frac{1}{t}\ge 8(1-2t)\iff 16t^2-8t+1\ge 0\iff (4t-1)^2\ge 0$.

Vậy ta có điều phải chứng minh. Dấu $=$ xảy ra tại $(a;b;c)=(\frac{1}{2};\frac{1}{2};0)$ và các hoán vị.

Ps: Phương pháp $p,q,r$ là một trong những phương pháp khá dùng đối với những bài toán mang tính đối xứng cao.


Bài viết đã được chỉnh sửa nội dung bởi tritanngo99: 26-08-2016 - 10:27


#75
tritanngo99

tritanngo99

    Đại úy

  • Điều hành viên THPT
  • 1644 Bài viết

Tiếp theo: 

Bài 39: Cho các số không âm thỏa mãn: $c=min(a;b;c)$. Chứng minh rằng:

$(\frac{a}{b-c})^2+(\frac{b}{c-a})^2\ge \frac{a^2+b^2+c^2}{ab+bc+ca}$.

Bài 40: Cho các số thực dương thỏa mãn: $a+b+c=1$. Tìm GTLN của:

$P=\sqrt{abc}(\frac{1}{a+bc}+\frac{1}{b+ac})-\frac{c}{c+ab}$. 



#76
anhquannbk

anhquannbk

    Sĩ quan

  • Thành viên
  • 477 Bài viết

Bài 40:

Bài này hướng đi giống bài 37.

Ta $ P=\sqrt{abc}(\dfrac{1}{a+bc}+\dfrac{1}{b+ac})-\dfrac{c}{c+ab}=\dfrac{\sqrt{abc}}{a+bc}+\dfrac{\sqrt{abc}}{b+ac}-\dfrac{c}{c+ab}=\dfrac{1}{\sqrt{\dfrac{a}{bc}}+\sqrt{\dfrac{bc}{a}}}+\dfrac{1}{\sqrt{\dfrac{b}{ac}}+\sqrt{\dfrac{ac}{b}}} -\dfrac{1}{1+\dfrac{ab}{c}}$

Đổi biến $ (x;y;z)=(\sqrt{\dfrac{bc}{a}},\sqrt{\dfrac{ac}{b}},\sqrt{\dfrac{ab}{c}}) $

Ta $ a+b+c=1 \Longrightarrow xy+yz+xz=1 $

Khi đó $ P=\dfrac{1}{x+\dfrac{1}{x}}+\dfrac{1}{y+\dfrac{1}{y}}-\dfrac{1}{1+z^{2}}=\dfrac{x}{1+x^2}+\dfrac{y}{1+y^2}-\dfrac{1}{1+z^2} $

Đặt $ x=tan\dfrac{A}{2} ,y=tan\dfrac{B}{2}, z=tan\dfrac{C}{2}$

Suy ra $ P=\dfrac{sinA+sinB}{2}-cos^2\dfrac{C}{2} $

Lại $ sinA+sinB=2sin(\dfrac{A+B}{2}).cos(\dfrac{A-B}{2}) \le 2cos\dfrac{C}{2} $

$ \iff P \le cos\dfrac{C}{2}-cos^2\dfrac{C}{2}=\dfrac{1}{4}-(cos\dfrac{C}{2}-\dfrac{1}{2})^2 \le \dfrac{1}{4} $

Vậy $ Max P=\dfrac{1}{4} $, xảy ra khi $ x=y=2-\sqrt{3} , z=\sqrt{3}$ $ \iff $ $ a=b=2\sqrt{3}-3,c=7-4\sqrt{3} $



#77
anhquannbk

anhquannbk

    Sĩ quan

  • Thành viên
  • 477 Bài viết

Bài 39:

Đặt $ f(a,b,c)=(\dfrac{a}{b-c})^2+(\dfrac{b}{a-c})^2-\dfrac{a^2+b^2+c^2}{ab+bc+ac} $

Nhận thấy dấu $ = $ xảy ra khi $ a=b, c=0 $

Ta sẽ dồn biến về biên. Không mất tính tổng quát, giả sử $ a=max(a,b,c) $.

Ta $ f(a+c,b,0)=(\dfrac{a+c}{b})^2+(\dfrac{b}{a+c})^2-\dfrac{(a+c)^2+b^2}{b(a+c)} $

Do $ c=min(a,b,c) $ nên $ a\ge c, b \ge c $

Xét $ f(a,b,c)-f(a+c,b,0)= (\dfrac{a}{b-c})^2-(\dfrac{a+c}{b})^2+(\dfrac{b}{a-c})^2-(\dfrac{b}{a+c})^2 + \dfrac{(a+c)^2+b^2}{b(a+c)}-\dfrac{a^2+b^2+c^2}{ab+bc+ac}$

Ta

$(\dfrac{a}{b-c})^2-(\dfrac{a+c}{b})^2 =(\dfrac{a}{b-c}+\dfrac{b}{a+c})(\dfrac{a}{b-c}-\dfrac{a+c}{b})=(\dfrac{a}{b-c}+\dfrac{b}{a+c}).\dfrac{c(a+c-b)}{b(b-c)} \ge 0$

$ (\dfrac{b}{a-c})^2-(\dfrac{b}{a+c})^2=(\dfrac{b}{a-c}+\dfrac{b}{a+c}).\dfrac{2bc}{(a-c)(a+c)} \ge 0 $

$ \dfrac{(a+c)^2+b^2}{b(a+c)}-\dfrac{a^2+b^2+c^2}{ab+bc+ac}=\dfrac{ac(a^2+b^2+c^2+2ab+2bc+2ac)}{b(a+c)(ab+bc+ac)} \ge 0 $

suy ra $ f(a,b,c) \ge f(a+c,b,0) $

Do đó ta chỉ cần chỉ ra $ f(a+c,b,0)\ge 0 $

tức

$ (\dfrac{a+c}{b})^2+(\dfrac{b}{a+c})^2-\dfrac{(a+c)^2+b^2}{b(a+c)} \ge 0 $

$ \iff (a+c)^4+b^4 \ge b(a+c)[(a+c)^2+b^2] $ $ \iff x^4+y^4 \ge xy(x^2+y^2) $ (đúng) với $ x=a+c, y=b $

Vậy BĐT được chứng minh, dấu bằng xảy ra khi $ a=b, c=0 $.



#78
tritanngo99

tritanngo99

    Đại úy

  • Điều hành viên THPT
  • 1644 Bài viết

Xin lỗi mọi người, Hôm nay mình quyết định đóng Topic tại đây và sẽ mở lại vào mùa hè năm sau.. Hẹn gặp lại mọi người.



#79
I Love MC

I Love MC

    Đại úy

  • Thành viên nổi bật 2016
  • 1861 Bài viết

$\frac{1}{a}+\frac{1}{b}+\frac{1}{c}\geq \frac{9}{a+b+c}$

$\frac{9}{a+b+c} + \frac{13}{a+b+c+1} \geq \frac{25}{4}$

đặt a +b+c=x ,bất đẳng thức cần chưng minh tương đương với

$\frac{9}{x}+\frac{13}{x+1}\geq \frac{25}{4}$ ,cái này phân tích ra thi cần chứng minh

$25x^{2} -63x -36 \geq 0$

$(25x+12)(x-3)\geq 0$

mà $x\geq 3$ suy ra điều chứng minh  :D


Bài viết đã được chỉnh sửa nội dung bởi I Love MC: 29-08-2016 - 17:09


#80
hoangpro1811

hoangpro1811

    Lính mới

  • Thành viên
  • 8 Bài viết

Giúp em nốt bài này nhé! Cho a,b,c>0,abc=1,tìm max của \sum \frac{a}{a+b^2+c^2}






0 người đang xem chủ đề

0 thành viên, 0 khách, 0 thành viên ẩn danh